Real Analysis, $limlimits_{nrightarrowinfty} int_{0}^{1} frac{e^{-nt}-(1-t)^n}{t} dt$












4












$begingroup$


I was trying to compute $limlimits_{nrightarrowinfty} int_{0}^{1} frac{e^{-nt}-(1-t)^n}{t} dt$ using Lebesgue's dominated convergence THM, but I can't exactly figure out how to do. I mean, I managed to prove that each the integrand function $f_n(t)$ is less or equal than $g_n(t)=e^{-nt}sqrt{n}e^frac{1}{sqrt{n}}, forall ninmathbb{N}$. And since we are dealing with positive functions and $int_{0}^{1} g_n(t)dtleqfrac{e}{sqrt{n}}overset{mathrm{nrightarrowinfty}}{rightarrow}0$, I can deduce that the original limit is $0$.





Now, I was just wondering if anyone is able to show analytically that there exists a function in $mathcal{L}^1$ which dominates all the $f_n$ in order to apply Lebesgue's dominated convergence THM. I made some attempts, but I failed.





Thanks in advance,
a humble half-mathematician.










share|cite|improve this question











$endgroup$

















    4












    $begingroup$


    I was trying to compute $limlimits_{nrightarrowinfty} int_{0}^{1} frac{e^{-nt}-(1-t)^n}{t} dt$ using Lebesgue's dominated convergence THM, but I can't exactly figure out how to do. I mean, I managed to prove that each the integrand function $f_n(t)$ is less or equal than $g_n(t)=e^{-nt}sqrt{n}e^frac{1}{sqrt{n}}, forall ninmathbb{N}$. And since we are dealing with positive functions and $int_{0}^{1} g_n(t)dtleqfrac{e}{sqrt{n}}overset{mathrm{nrightarrowinfty}}{rightarrow}0$, I can deduce that the original limit is $0$.





    Now, I was just wondering if anyone is able to show analytically that there exists a function in $mathcal{L}^1$ which dominates all the $f_n$ in order to apply Lebesgue's dominated convergence THM. I made some attempts, but I failed.





    Thanks in advance,
    a humble half-mathematician.










    share|cite|improve this question











    $endgroup$















      4












      4








      4


      0



      $begingroup$


      I was trying to compute $limlimits_{nrightarrowinfty} int_{0}^{1} frac{e^{-nt}-(1-t)^n}{t} dt$ using Lebesgue's dominated convergence THM, but I can't exactly figure out how to do. I mean, I managed to prove that each the integrand function $f_n(t)$ is less or equal than $g_n(t)=e^{-nt}sqrt{n}e^frac{1}{sqrt{n}}, forall ninmathbb{N}$. And since we are dealing with positive functions and $int_{0}^{1} g_n(t)dtleqfrac{e}{sqrt{n}}overset{mathrm{nrightarrowinfty}}{rightarrow}0$, I can deduce that the original limit is $0$.





      Now, I was just wondering if anyone is able to show analytically that there exists a function in $mathcal{L}^1$ which dominates all the $f_n$ in order to apply Lebesgue's dominated convergence THM. I made some attempts, but I failed.





      Thanks in advance,
      a humble half-mathematician.










      share|cite|improve this question











      $endgroup$




      I was trying to compute $limlimits_{nrightarrowinfty} int_{0}^{1} frac{e^{-nt}-(1-t)^n}{t} dt$ using Lebesgue's dominated convergence THM, but I can't exactly figure out how to do. I mean, I managed to prove that each the integrand function $f_n(t)$ is less or equal than $g_n(t)=e^{-nt}sqrt{n}e^frac{1}{sqrt{n}}, forall ninmathbb{N}$. And since we are dealing with positive functions and $int_{0}^{1} g_n(t)dtleqfrac{e}{sqrt{n}}overset{mathrm{nrightarrowinfty}}{rightarrow}0$, I can deduce that the original limit is $0$.





      Now, I was just wondering if anyone is able to show analytically that there exists a function in $mathcal{L}^1$ which dominates all the $f_n$ in order to apply Lebesgue's dominated convergence THM. I made some attempts, but I failed.





      Thanks in advance,
      a humble half-mathematician.







      real-analysis integration limits convergence definite-integrals






      share|cite|improve this question















      share|cite|improve this question













      share|cite|improve this question




      share|cite|improve this question








      edited Dec 8 '18 at 11:46









      Batominovski

      33k33293




      33k33293










      asked Dec 8 '18 at 10:24









      piotorpiotor

      405




      405






















          1 Answer
          1






          active

          oldest

          votes


















          3












          $begingroup$

          We have
          $$exp(-nt)-(1-t)^n=exp(-nt),Big(1-big((1-t)exp(t)big)^nBig),.tag{*}$$
          By Bernoulli's Inequaliy,
          $$big((1-t)exp(t)big)^ngeq 1+n,big((1-t)exp(t)-1big),.$$
          Therefore,
          $$exp(-nt)-(1-t)^nleq n,exp(-nt),big(1-(1-t)exp(t)big),.$$
          By taking derivative with respect to $n$, we can show that
          $$n,exp(-nt)leq frac{1}{text{e},t}text{ for all }t>0text{ and positive integers }n,.$$
          Furthermore, we have
          $$(1-t),exp(t)=1-sum_{k=1}^infty,left(frac{k-1}{k!}right),t^kgeq 1-t^2,sum_{k=2}^infty,frac{k-1}{k!}=1-t^2$$
          for all $tin[0,1]$.



          By (*), we conclude that
          $$f_n(t):=frac{exp(-nt)-(1-t)^n}{t}leq frac{1}{text{e},t^2},big(1-(1-t^2)big)=frac{1}{text{e}}$$
          for every $tin (0,1]$ and every positive integer $n$ (where the only equality case is when $n=1$ and $t=1$). Therefore, the constant function $fequiv dfrac{1}{text{e}}$ dominates $f_n$ for all $n=1,2,3,ldots$.






          share|cite|improve this answer









          $endgroup$













          • $begingroup$
            Absolutely brilliant. Thanks a lot.
            $endgroup$
            – piotor
            Dec 8 '18 at 18:45











          Your Answer





          StackExchange.ifUsing("editor", function () {
          return StackExchange.using("mathjaxEditing", function () {
          StackExchange.MarkdownEditor.creationCallbacks.add(function (editor, postfix) {
          StackExchange.mathjaxEditing.prepareWmdForMathJax(editor, postfix, [["$", "$"], ["\\(","\\)"]]);
          });
          });
          }, "mathjax-editing");

          StackExchange.ready(function() {
          var channelOptions = {
          tags: "".split(" "),
          id: "69"
          };
          initTagRenderer("".split(" "), "".split(" "), channelOptions);

          StackExchange.using("externalEditor", function() {
          // Have to fire editor after snippets, if snippets enabled
          if (StackExchange.settings.snippets.snippetsEnabled) {
          StackExchange.using("snippets", function() {
          createEditor();
          });
          }
          else {
          createEditor();
          }
          });

          function createEditor() {
          StackExchange.prepareEditor({
          heartbeatType: 'answer',
          autoActivateHeartbeat: false,
          convertImagesToLinks: true,
          noModals: true,
          showLowRepImageUploadWarning: true,
          reputationToPostImages: 10,
          bindNavPrevention: true,
          postfix: "",
          imageUploader: {
          brandingHtml: "Powered by u003ca class="icon-imgur-white" href="https://imgur.com/"u003eu003c/au003e",
          contentPolicyHtml: "User contributions licensed under u003ca href="https://creativecommons.org/licenses/by-sa/3.0/"u003ecc by-sa 3.0 with attribution requiredu003c/au003e u003ca href="https://stackoverflow.com/legal/content-policy"u003e(content policy)u003c/au003e",
          allowUrls: true
          },
          noCode: true, onDemand: true,
          discardSelector: ".discard-answer"
          ,immediatelyShowMarkdownHelp:true
          });


          }
          });














          draft saved

          draft discarded


















          StackExchange.ready(
          function () {
          StackExchange.openid.initPostLogin('.new-post-login', 'https%3a%2f%2fmath.stackexchange.com%2fquestions%2f3030926%2freal-analysis-lim-limits-n-rightarrow-infty-int-01-frace-nt-1-t%23new-answer', 'question_page');
          }
          );

          Post as a guest















          Required, but never shown

























          1 Answer
          1






          active

          oldest

          votes








          1 Answer
          1






          active

          oldest

          votes









          active

          oldest

          votes






          active

          oldest

          votes









          3












          $begingroup$

          We have
          $$exp(-nt)-(1-t)^n=exp(-nt),Big(1-big((1-t)exp(t)big)^nBig),.tag{*}$$
          By Bernoulli's Inequaliy,
          $$big((1-t)exp(t)big)^ngeq 1+n,big((1-t)exp(t)-1big),.$$
          Therefore,
          $$exp(-nt)-(1-t)^nleq n,exp(-nt),big(1-(1-t)exp(t)big),.$$
          By taking derivative with respect to $n$, we can show that
          $$n,exp(-nt)leq frac{1}{text{e},t}text{ for all }t>0text{ and positive integers }n,.$$
          Furthermore, we have
          $$(1-t),exp(t)=1-sum_{k=1}^infty,left(frac{k-1}{k!}right),t^kgeq 1-t^2,sum_{k=2}^infty,frac{k-1}{k!}=1-t^2$$
          for all $tin[0,1]$.



          By (*), we conclude that
          $$f_n(t):=frac{exp(-nt)-(1-t)^n}{t}leq frac{1}{text{e},t^2},big(1-(1-t^2)big)=frac{1}{text{e}}$$
          for every $tin (0,1]$ and every positive integer $n$ (where the only equality case is when $n=1$ and $t=1$). Therefore, the constant function $fequiv dfrac{1}{text{e}}$ dominates $f_n$ for all $n=1,2,3,ldots$.






          share|cite|improve this answer









          $endgroup$













          • $begingroup$
            Absolutely brilliant. Thanks a lot.
            $endgroup$
            – piotor
            Dec 8 '18 at 18:45
















          3












          $begingroup$

          We have
          $$exp(-nt)-(1-t)^n=exp(-nt),Big(1-big((1-t)exp(t)big)^nBig),.tag{*}$$
          By Bernoulli's Inequaliy,
          $$big((1-t)exp(t)big)^ngeq 1+n,big((1-t)exp(t)-1big),.$$
          Therefore,
          $$exp(-nt)-(1-t)^nleq n,exp(-nt),big(1-(1-t)exp(t)big),.$$
          By taking derivative with respect to $n$, we can show that
          $$n,exp(-nt)leq frac{1}{text{e},t}text{ for all }t>0text{ and positive integers }n,.$$
          Furthermore, we have
          $$(1-t),exp(t)=1-sum_{k=1}^infty,left(frac{k-1}{k!}right),t^kgeq 1-t^2,sum_{k=2}^infty,frac{k-1}{k!}=1-t^2$$
          for all $tin[0,1]$.



          By (*), we conclude that
          $$f_n(t):=frac{exp(-nt)-(1-t)^n}{t}leq frac{1}{text{e},t^2},big(1-(1-t^2)big)=frac{1}{text{e}}$$
          for every $tin (0,1]$ and every positive integer $n$ (where the only equality case is when $n=1$ and $t=1$). Therefore, the constant function $fequiv dfrac{1}{text{e}}$ dominates $f_n$ for all $n=1,2,3,ldots$.






          share|cite|improve this answer









          $endgroup$













          • $begingroup$
            Absolutely brilliant. Thanks a lot.
            $endgroup$
            – piotor
            Dec 8 '18 at 18:45














          3












          3








          3





          $begingroup$

          We have
          $$exp(-nt)-(1-t)^n=exp(-nt),Big(1-big((1-t)exp(t)big)^nBig),.tag{*}$$
          By Bernoulli's Inequaliy,
          $$big((1-t)exp(t)big)^ngeq 1+n,big((1-t)exp(t)-1big),.$$
          Therefore,
          $$exp(-nt)-(1-t)^nleq n,exp(-nt),big(1-(1-t)exp(t)big),.$$
          By taking derivative with respect to $n$, we can show that
          $$n,exp(-nt)leq frac{1}{text{e},t}text{ for all }t>0text{ and positive integers }n,.$$
          Furthermore, we have
          $$(1-t),exp(t)=1-sum_{k=1}^infty,left(frac{k-1}{k!}right),t^kgeq 1-t^2,sum_{k=2}^infty,frac{k-1}{k!}=1-t^2$$
          for all $tin[0,1]$.



          By (*), we conclude that
          $$f_n(t):=frac{exp(-nt)-(1-t)^n}{t}leq frac{1}{text{e},t^2},big(1-(1-t^2)big)=frac{1}{text{e}}$$
          for every $tin (0,1]$ and every positive integer $n$ (where the only equality case is when $n=1$ and $t=1$). Therefore, the constant function $fequiv dfrac{1}{text{e}}$ dominates $f_n$ for all $n=1,2,3,ldots$.






          share|cite|improve this answer









          $endgroup$



          We have
          $$exp(-nt)-(1-t)^n=exp(-nt),Big(1-big((1-t)exp(t)big)^nBig),.tag{*}$$
          By Bernoulli's Inequaliy,
          $$big((1-t)exp(t)big)^ngeq 1+n,big((1-t)exp(t)-1big),.$$
          Therefore,
          $$exp(-nt)-(1-t)^nleq n,exp(-nt),big(1-(1-t)exp(t)big),.$$
          By taking derivative with respect to $n$, we can show that
          $$n,exp(-nt)leq frac{1}{text{e},t}text{ for all }t>0text{ and positive integers }n,.$$
          Furthermore, we have
          $$(1-t),exp(t)=1-sum_{k=1}^infty,left(frac{k-1}{k!}right),t^kgeq 1-t^2,sum_{k=2}^infty,frac{k-1}{k!}=1-t^2$$
          for all $tin[0,1]$.



          By (*), we conclude that
          $$f_n(t):=frac{exp(-nt)-(1-t)^n}{t}leq frac{1}{text{e},t^2},big(1-(1-t^2)big)=frac{1}{text{e}}$$
          for every $tin (0,1]$ and every positive integer $n$ (where the only equality case is when $n=1$ and $t=1$). Therefore, the constant function $fequiv dfrac{1}{text{e}}$ dominates $f_n$ for all $n=1,2,3,ldots$.







          share|cite|improve this answer












          share|cite|improve this answer



          share|cite|improve this answer










          answered Dec 8 '18 at 11:39









          BatominovskiBatominovski

          33k33293




          33k33293












          • $begingroup$
            Absolutely brilliant. Thanks a lot.
            $endgroup$
            – piotor
            Dec 8 '18 at 18:45


















          • $begingroup$
            Absolutely brilliant. Thanks a lot.
            $endgroup$
            – piotor
            Dec 8 '18 at 18:45
















          $begingroup$
          Absolutely brilliant. Thanks a lot.
          $endgroup$
          – piotor
          Dec 8 '18 at 18:45




          $begingroup$
          Absolutely brilliant. Thanks a lot.
          $endgroup$
          – piotor
          Dec 8 '18 at 18:45


















          draft saved

          draft discarded




















































          Thanks for contributing an answer to Mathematics Stack Exchange!


          • Please be sure to answer the question. Provide details and share your research!

          But avoid



          • Asking for help, clarification, or responding to other answers.

          • Making statements based on opinion; back them up with references or personal experience.


          Use MathJax to format equations. MathJax reference.


          To learn more, see our tips on writing great answers.




          draft saved


          draft discarded














          StackExchange.ready(
          function () {
          StackExchange.openid.initPostLogin('.new-post-login', 'https%3a%2f%2fmath.stackexchange.com%2fquestions%2f3030926%2freal-analysis-lim-limits-n-rightarrow-infty-int-01-frace-nt-1-t%23new-answer', 'question_page');
          }
          );

          Post as a guest















          Required, but never shown





















































          Required, but never shown














          Required, but never shown












          Required, but never shown







          Required, but never shown

































          Required, but never shown














          Required, but never shown












          Required, but never shown







          Required, but never shown







          Popular posts from this blog

          Plaza Victoria

          In PowerPoint, is there a keyboard shortcut for bulleted / numbered list?

          How to put 3 figures in Latex with 2 figures side by side and 1 below these side by side images but in...